Change order of integration to obtain equivalent integrals
$begingroup$
In the textbook Thermal physics, by Garg, Bansal and Ghosh [1], there exist two integrals on page 640, which are claimed to be equivalent (without demonstration) by a change in the order of integration.
begin{align}
I_1&=int_{0}^{infty}dv_1e^{-frac{mv_1^2}{2k_BT}}v_1^2int_{0}^{v_1}dv_2e^{-frac{mv_2^2}{2k_BT}}left(frac{3v_1^2+v_2^2}{3v_1}right)v_2^2\
I_2=&int_{0}^{infty}dv_1e^{-frac{mv_1^2}{2k_BT}}v_1^2int_{v_1}^{infty}dv_2e^{-frac{mv_2^2}{2k_BT}}left(frac{3v_2^2+v_1^2}{3v_2}right)v_2^2
end{align}
It is stated that an interchange of the order of integration will show them to be equivalent. To show this, it must be done in such a way that the limits over $v_1$ in $I_1$, change from $0$ to $infty$ into $v_1$ to $infty$.
However, when attempting this problem myself I notice two things. The change in limits on $I_1$ doesn't appear to yield an equivalent region to integrate over, and even if it did, the functions to be integrated are not the same. This indicates to me that there is some other trick at play.
So the question follows: how can I prove these two integrals are the
same, and what other technique should be employed, besides a change in
integration, in order to show it?
To combine the integrals into something I can change the order in, I have been relabeling the first $v_1$ as $x$, then writing something like this:
begin{align}
I_1&=int_{0}^{v_1}int_{0}^{infty}e^{-frac{mx^2}{2k_BT}}x^2e^{-frac{mv_2^2}{2k_BT}}left(frac{3v_1^2+v_2^2}{3v_1}right)v_2^2dxdv_2\
end{align}
I'm asking this question because I promised @Thorondor I'd show them to be equivalent in this question [2], but unfortunately I got stumped.
References:
[1] Garg S.C. Bansal R.M. Ghosh C.K. (2012) Thermal Physics: Kinetic Theory, Thermodynamics and Statistical Mechanics. Tata McGraw Hill Education, New Delhi, 638-641.
[2] https://physics.stackexchange.com/questions/448761/rigorous-derivation-of-the-mean-free-path-in-a-gas
integration physics
$endgroup$
add a comment |
$begingroup$
In the textbook Thermal physics, by Garg, Bansal and Ghosh [1], there exist two integrals on page 640, which are claimed to be equivalent (without demonstration) by a change in the order of integration.
begin{align}
I_1&=int_{0}^{infty}dv_1e^{-frac{mv_1^2}{2k_BT}}v_1^2int_{0}^{v_1}dv_2e^{-frac{mv_2^2}{2k_BT}}left(frac{3v_1^2+v_2^2}{3v_1}right)v_2^2\
I_2=&int_{0}^{infty}dv_1e^{-frac{mv_1^2}{2k_BT}}v_1^2int_{v_1}^{infty}dv_2e^{-frac{mv_2^2}{2k_BT}}left(frac{3v_2^2+v_1^2}{3v_2}right)v_2^2
end{align}
It is stated that an interchange of the order of integration will show them to be equivalent. To show this, it must be done in such a way that the limits over $v_1$ in $I_1$, change from $0$ to $infty$ into $v_1$ to $infty$.
However, when attempting this problem myself I notice two things. The change in limits on $I_1$ doesn't appear to yield an equivalent region to integrate over, and even if it did, the functions to be integrated are not the same. This indicates to me that there is some other trick at play.
So the question follows: how can I prove these two integrals are the
same, and what other technique should be employed, besides a change in
integration, in order to show it?
To combine the integrals into something I can change the order in, I have been relabeling the first $v_1$ as $x$, then writing something like this:
begin{align}
I_1&=int_{0}^{v_1}int_{0}^{infty}e^{-frac{mx^2}{2k_BT}}x^2e^{-frac{mv_2^2}{2k_BT}}left(frac{3v_1^2+v_2^2}{3v_1}right)v_2^2dxdv_2\
end{align}
I'm asking this question because I promised @Thorondor I'd show them to be equivalent in this question [2], but unfortunately I got stumped.
References:
[1] Garg S.C. Bansal R.M. Ghosh C.K. (2012) Thermal Physics: Kinetic Theory, Thermodynamics and Statistical Mechanics. Tata McGraw Hill Education, New Delhi, 638-641.
[2] https://physics.stackexchange.com/questions/448761/rigorous-derivation-of-the-mean-free-path-in-a-gas
integration physics
$endgroup$
add a comment |
$begingroup$
In the textbook Thermal physics, by Garg, Bansal and Ghosh [1], there exist two integrals on page 640, which are claimed to be equivalent (without demonstration) by a change in the order of integration.
begin{align}
I_1&=int_{0}^{infty}dv_1e^{-frac{mv_1^2}{2k_BT}}v_1^2int_{0}^{v_1}dv_2e^{-frac{mv_2^2}{2k_BT}}left(frac{3v_1^2+v_2^2}{3v_1}right)v_2^2\
I_2=&int_{0}^{infty}dv_1e^{-frac{mv_1^2}{2k_BT}}v_1^2int_{v_1}^{infty}dv_2e^{-frac{mv_2^2}{2k_BT}}left(frac{3v_2^2+v_1^2}{3v_2}right)v_2^2
end{align}
It is stated that an interchange of the order of integration will show them to be equivalent. To show this, it must be done in such a way that the limits over $v_1$ in $I_1$, change from $0$ to $infty$ into $v_1$ to $infty$.
However, when attempting this problem myself I notice two things. The change in limits on $I_1$ doesn't appear to yield an equivalent region to integrate over, and even if it did, the functions to be integrated are not the same. This indicates to me that there is some other trick at play.
So the question follows: how can I prove these two integrals are the
same, and what other technique should be employed, besides a change in
integration, in order to show it?
To combine the integrals into something I can change the order in, I have been relabeling the first $v_1$ as $x$, then writing something like this:
begin{align}
I_1&=int_{0}^{v_1}int_{0}^{infty}e^{-frac{mx^2}{2k_BT}}x^2e^{-frac{mv_2^2}{2k_BT}}left(frac{3v_1^2+v_2^2}{3v_1}right)v_2^2dxdv_2\
end{align}
I'm asking this question because I promised @Thorondor I'd show them to be equivalent in this question [2], but unfortunately I got stumped.
References:
[1] Garg S.C. Bansal R.M. Ghosh C.K. (2012) Thermal Physics: Kinetic Theory, Thermodynamics and Statistical Mechanics. Tata McGraw Hill Education, New Delhi, 638-641.
[2] https://physics.stackexchange.com/questions/448761/rigorous-derivation-of-the-mean-free-path-in-a-gas
integration physics
$endgroup$
In the textbook Thermal physics, by Garg, Bansal and Ghosh [1], there exist two integrals on page 640, which are claimed to be equivalent (without demonstration) by a change in the order of integration.
begin{align}
I_1&=int_{0}^{infty}dv_1e^{-frac{mv_1^2}{2k_BT}}v_1^2int_{0}^{v_1}dv_2e^{-frac{mv_2^2}{2k_BT}}left(frac{3v_1^2+v_2^2}{3v_1}right)v_2^2\
I_2=&int_{0}^{infty}dv_1e^{-frac{mv_1^2}{2k_BT}}v_1^2int_{v_1}^{infty}dv_2e^{-frac{mv_2^2}{2k_BT}}left(frac{3v_2^2+v_1^2}{3v_2}right)v_2^2
end{align}
It is stated that an interchange of the order of integration will show them to be equivalent. To show this, it must be done in such a way that the limits over $v_1$ in $I_1$, change from $0$ to $infty$ into $v_1$ to $infty$.
However, when attempting this problem myself I notice two things. The change in limits on $I_1$ doesn't appear to yield an equivalent region to integrate over, and even if it did, the functions to be integrated are not the same. This indicates to me that there is some other trick at play.
So the question follows: how can I prove these two integrals are the
same, and what other technique should be employed, besides a change in
integration, in order to show it?
To combine the integrals into something I can change the order in, I have been relabeling the first $v_1$ as $x$, then writing something like this:
begin{align}
I_1&=int_{0}^{v_1}int_{0}^{infty}e^{-frac{mx^2}{2k_BT}}x^2e^{-frac{mv_2^2}{2k_BT}}left(frac{3v_1^2+v_2^2}{3v_1}right)v_2^2dxdv_2\
end{align}
I'm asking this question because I promised @Thorondor I'd show them to be equivalent in this question [2], but unfortunately I got stumped.
References:
[1] Garg S.C. Bansal R.M. Ghosh C.K. (2012) Thermal Physics: Kinetic Theory, Thermodynamics and Statistical Mechanics. Tata McGraw Hill Education, New Delhi, 638-641.
[2] https://physics.stackexchange.com/questions/448761/rigorous-derivation-of-the-mean-free-path-in-a-gas
integration physics
integration physics
asked Jan 2 at 5:14
user400188user400188
4291614
4291614
add a comment |
add a comment |
1 Answer
1
active
oldest
votes
$begingroup$
Write the inner integral of $I_1$ as
$$int_{0}^{infty}dv_2e^{-frac{mv_2^2}{2k_BT}}left(frac{3v_1^2+v_2^2}{3v_1}right)v_2^2 mathbf{1}_{{v_1 geqslant v_2}}$$
where
$$mathbf{1}_{{v_1 geqslant v_2}} = begin{cases}1, & v_1 geqslant v_2 \ 0 , & v_1 < v_2end{cases}$$
Apply Fubini's theorem to interchange integrals noting that
$$int_{0}^{infty}dv_1f(v_1,v_2)mathbf{1}_{{v_1 geqslant v_2}} = int_{v_2}^{infty}dv_1f(v_1,v_2) $$
and then change notation $v_1 iff v_2$.
$endgroup$
$begingroup$
Would it be correct to say here, that we are integrating over different regions, but the areas of both regions are the same, justifying the change in notation? I'm just trying to verify if my understanding of the answer is correct.
$endgroup$
– user400188
Jan 2 at 5:46
1
$begingroup$
If you follow my steps you get $I_1 = int_{0}^{infty}dv_2e^{-frac{mv_2^2}{2k_BT}}v_2^2int_{v_2}^{infty}dv_1e^{-frac{mv_1^2}{2k_BT}}left(frac{3v_1^2+v_2^2}{3v_1}right)v_1^2$ before changing notation. I think you can see why this works just by looking at a graph of the original region of integration -- an unbounded triangle-like region. Now switching variable names requires no deep mathematical justification for a definite integral.
$endgroup$
– RRL
Jan 2 at 5:55
$begingroup$
Thanks @RRL, I drew the graphs earlier and wanted to confirm my suspicion.
$endgroup$
– user400188
Jan 2 at 5:56
add a comment |
Your Answer
StackExchange.ready(function() {
var channelOptions = {
tags: "".split(" "),
id: "69"
};
initTagRenderer("".split(" "), "".split(" "), channelOptions);
StackExchange.using("externalEditor", function() {
// Have to fire editor after snippets, if snippets enabled
if (StackExchange.settings.snippets.snippetsEnabled) {
StackExchange.using("snippets", function() {
createEditor();
});
}
else {
createEditor();
}
});
function createEditor() {
StackExchange.prepareEditor({
heartbeatType: 'answer',
autoActivateHeartbeat: false,
convertImagesToLinks: true,
noModals: true,
showLowRepImageUploadWarning: true,
reputationToPostImages: 10,
bindNavPrevention: true,
postfix: "",
imageUploader: {
brandingHtml: "Powered by u003ca class="icon-imgur-white" href="https://imgur.com/"u003eu003c/au003e",
contentPolicyHtml: "User contributions licensed under u003ca href="https://creativecommons.org/licenses/by-sa/3.0/"u003ecc by-sa 3.0 with attribution requiredu003c/au003e u003ca href="https://stackoverflow.com/legal/content-policy"u003e(content policy)u003c/au003e",
allowUrls: true
},
noCode: true, onDemand: true,
discardSelector: ".discard-answer"
,immediatelyShowMarkdownHelp:true
});
}
});
Sign up or log in
StackExchange.ready(function () {
StackExchange.helpers.onClickDraftSave('#login-link');
});
Sign up using Google
Sign up using Facebook
Sign up using Email and Password
Post as a guest
Required, but never shown
StackExchange.ready(
function () {
StackExchange.openid.initPostLogin('.new-post-login', 'https%3a%2f%2fmath.stackexchange.com%2fquestions%2f3059156%2fchange-order-of-integration-to-obtain-equivalent-integrals%23new-answer', 'question_page');
}
);
Post as a guest
Required, but never shown
1 Answer
1
active
oldest
votes
1 Answer
1
active
oldest
votes
active
oldest
votes
active
oldest
votes
$begingroup$
Write the inner integral of $I_1$ as
$$int_{0}^{infty}dv_2e^{-frac{mv_2^2}{2k_BT}}left(frac{3v_1^2+v_2^2}{3v_1}right)v_2^2 mathbf{1}_{{v_1 geqslant v_2}}$$
where
$$mathbf{1}_{{v_1 geqslant v_2}} = begin{cases}1, & v_1 geqslant v_2 \ 0 , & v_1 < v_2end{cases}$$
Apply Fubini's theorem to interchange integrals noting that
$$int_{0}^{infty}dv_1f(v_1,v_2)mathbf{1}_{{v_1 geqslant v_2}} = int_{v_2}^{infty}dv_1f(v_1,v_2) $$
and then change notation $v_1 iff v_2$.
$endgroup$
$begingroup$
Would it be correct to say here, that we are integrating over different regions, but the areas of both regions are the same, justifying the change in notation? I'm just trying to verify if my understanding of the answer is correct.
$endgroup$
– user400188
Jan 2 at 5:46
1
$begingroup$
If you follow my steps you get $I_1 = int_{0}^{infty}dv_2e^{-frac{mv_2^2}{2k_BT}}v_2^2int_{v_2}^{infty}dv_1e^{-frac{mv_1^2}{2k_BT}}left(frac{3v_1^2+v_2^2}{3v_1}right)v_1^2$ before changing notation. I think you can see why this works just by looking at a graph of the original region of integration -- an unbounded triangle-like region. Now switching variable names requires no deep mathematical justification for a definite integral.
$endgroup$
– RRL
Jan 2 at 5:55
$begingroup$
Thanks @RRL, I drew the graphs earlier and wanted to confirm my suspicion.
$endgroup$
– user400188
Jan 2 at 5:56
add a comment |
$begingroup$
Write the inner integral of $I_1$ as
$$int_{0}^{infty}dv_2e^{-frac{mv_2^2}{2k_BT}}left(frac{3v_1^2+v_2^2}{3v_1}right)v_2^2 mathbf{1}_{{v_1 geqslant v_2}}$$
where
$$mathbf{1}_{{v_1 geqslant v_2}} = begin{cases}1, & v_1 geqslant v_2 \ 0 , & v_1 < v_2end{cases}$$
Apply Fubini's theorem to interchange integrals noting that
$$int_{0}^{infty}dv_1f(v_1,v_2)mathbf{1}_{{v_1 geqslant v_2}} = int_{v_2}^{infty}dv_1f(v_1,v_2) $$
and then change notation $v_1 iff v_2$.
$endgroup$
$begingroup$
Would it be correct to say here, that we are integrating over different regions, but the areas of both regions are the same, justifying the change in notation? I'm just trying to verify if my understanding of the answer is correct.
$endgroup$
– user400188
Jan 2 at 5:46
1
$begingroup$
If you follow my steps you get $I_1 = int_{0}^{infty}dv_2e^{-frac{mv_2^2}{2k_BT}}v_2^2int_{v_2}^{infty}dv_1e^{-frac{mv_1^2}{2k_BT}}left(frac{3v_1^2+v_2^2}{3v_1}right)v_1^2$ before changing notation. I think you can see why this works just by looking at a graph of the original region of integration -- an unbounded triangle-like region. Now switching variable names requires no deep mathematical justification for a definite integral.
$endgroup$
– RRL
Jan 2 at 5:55
$begingroup$
Thanks @RRL, I drew the graphs earlier and wanted to confirm my suspicion.
$endgroup$
– user400188
Jan 2 at 5:56
add a comment |
$begingroup$
Write the inner integral of $I_1$ as
$$int_{0}^{infty}dv_2e^{-frac{mv_2^2}{2k_BT}}left(frac{3v_1^2+v_2^2}{3v_1}right)v_2^2 mathbf{1}_{{v_1 geqslant v_2}}$$
where
$$mathbf{1}_{{v_1 geqslant v_2}} = begin{cases}1, & v_1 geqslant v_2 \ 0 , & v_1 < v_2end{cases}$$
Apply Fubini's theorem to interchange integrals noting that
$$int_{0}^{infty}dv_1f(v_1,v_2)mathbf{1}_{{v_1 geqslant v_2}} = int_{v_2}^{infty}dv_1f(v_1,v_2) $$
and then change notation $v_1 iff v_2$.
$endgroup$
Write the inner integral of $I_1$ as
$$int_{0}^{infty}dv_2e^{-frac{mv_2^2}{2k_BT}}left(frac{3v_1^2+v_2^2}{3v_1}right)v_2^2 mathbf{1}_{{v_1 geqslant v_2}}$$
where
$$mathbf{1}_{{v_1 geqslant v_2}} = begin{cases}1, & v_1 geqslant v_2 \ 0 , & v_1 < v_2end{cases}$$
Apply Fubini's theorem to interchange integrals noting that
$$int_{0}^{infty}dv_1f(v_1,v_2)mathbf{1}_{{v_1 geqslant v_2}} = int_{v_2}^{infty}dv_1f(v_1,v_2) $$
and then change notation $v_1 iff v_2$.
answered Jan 2 at 5:37
RRLRRL
53.9k52675
53.9k52675
$begingroup$
Would it be correct to say here, that we are integrating over different regions, but the areas of both regions are the same, justifying the change in notation? I'm just trying to verify if my understanding of the answer is correct.
$endgroup$
– user400188
Jan 2 at 5:46
1
$begingroup$
If you follow my steps you get $I_1 = int_{0}^{infty}dv_2e^{-frac{mv_2^2}{2k_BT}}v_2^2int_{v_2}^{infty}dv_1e^{-frac{mv_1^2}{2k_BT}}left(frac{3v_1^2+v_2^2}{3v_1}right)v_1^2$ before changing notation. I think you can see why this works just by looking at a graph of the original region of integration -- an unbounded triangle-like region. Now switching variable names requires no deep mathematical justification for a definite integral.
$endgroup$
– RRL
Jan 2 at 5:55
$begingroup$
Thanks @RRL, I drew the graphs earlier and wanted to confirm my suspicion.
$endgroup$
– user400188
Jan 2 at 5:56
add a comment |
$begingroup$
Would it be correct to say here, that we are integrating over different regions, but the areas of both regions are the same, justifying the change in notation? I'm just trying to verify if my understanding of the answer is correct.
$endgroup$
– user400188
Jan 2 at 5:46
1
$begingroup$
If you follow my steps you get $I_1 = int_{0}^{infty}dv_2e^{-frac{mv_2^2}{2k_BT}}v_2^2int_{v_2}^{infty}dv_1e^{-frac{mv_1^2}{2k_BT}}left(frac{3v_1^2+v_2^2}{3v_1}right)v_1^2$ before changing notation. I think you can see why this works just by looking at a graph of the original region of integration -- an unbounded triangle-like region. Now switching variable names requires no deep mathematical justification for a definite integral.
$endgroup$
– RRL
Jan 2 at 5:55
$begingroup$
Thanks @RRL, I drew the graphs earlier and wanted to confirm my suspicion.
$endgroup$
– user400188
Jan 2 at 5:56
$begingroup$
Would it be correct to say here, that we are integrating over different regions, but the areas of both regions are the same, justifying the change in notation? I'm just trying to verify if my understanding of the answer is correct.
$endgroup$
– user400188
Jan 2 at 5:46
$begingroup$
Would it be correct to say here, that we are integrating over different regions, but the areas of both regions are the same, justifying the change in notation? I'm just trying to verify if my understanding of the answer is correct.
$endgroup$
– user400188
Jan 2 at 5:46
1
1
$begingroup$
If you follow my steps you get $I_1 = int_{0}^{infty}dv_2e^{-frac{mv_2^2}{2k_BT}}v_2^2int_{v_2}^{infty}dv_1e^{-frac{mv_1^2}{2k_BT}}left(frac{3v_1^2+v_2^2}{3v_1}right)v_1^2$ before changing notation. I think you can see why this works just by looking at a graph of the original region of integration -- an unbounded triangle-like region. Now switching variable names requires no deep mathematical justification for a definite integral.
$endgroup$
– RRL
Jan 2 at 5:55
$begingroup$
If you follow my steps you get $I_1 = int_{0}^{infty}dv_2e^{-frac{mv_2^2}{2k_BT}}v_2^2int_{v_2}^{infty}dv_1e^{-frac{mv_1^2}{2k_BT}}left(frac{3v_1^2+v_2^2}{3v_1}right)v_1^2$ before changing notation. I think you can see why this works just by looking at a graph of the original region of integration -- an unbounded triangle-like region. Now switching variable names requires no deep mathematical justification for a definite integral.
$endgroup$
– RRL
Jan 2 at 5:55
$begingroup$
Thanks @RRL, I drew the graphs earlier and wanted to confirm my suspicion.
$endgroup$
– user400188
Jan 2 at 5:56
$begingroup$
Thanks @RRL, I drew the graphs earlier and wanted to confirm my suspicion.
$endgroup$
– user400188
Jan 2 at 5:56
add a comment |
Thanks for contributing an answer to Mathematics Stack Exchange!
- Please be sure to answer the question. Provide details and share your research!
But avoid …
- Asking for help, clarification, or responding to other answers.
- Making statements based on opinion; back them up with references or personal experience.
Use MathJax to format equations. MathJax reference.
To learn more, see our tips on writing great answers.
Sign up or log in
StackExchange.ready(function () {
StackExchange.helpers.onClickDraftSave('#login-link');
});
Sign up using Google
Sign up using Facebook
Sign up using Email and Password
Post as a guest
Required, but never shown
StackExchange.ready(
function () {
StackExchange.openid.initPostLogin('.new-post-login', 'https%3a%2f%2fmath.stackexchange.com%2fquestions%2f3059156%2fchange-order-of-integration-to-obtain-equivalent-integrals%23new-answer', 'question_page');
}
);
Post as a guest
Required, but never shown
Sign up or log in
StackExchange.ready(function () {
StackExchange.helpers.onClickDraftSave('#login-link');
});
Sign up using Google
Sign up using Facebook
Sign up using Email and Password
Post as a guest
Required, but never shown
Sign up or log in
StackExchange.ready(function () {
StackExchange.helpers.onClickDraftSave('#login-link');
});
Sign up using Google
Sign up using Facebook
Sign up using Email and Password
Post as a guest
Required, but never shown
Sign up or log in
StackExchange.ready(function () {
StackExchange.helpers.onClickDraftSave('#login-link');
});
Sign up using Google
Sign up using Facebook
Sign up using Email and Password
Sign up using Google
Sign up using Facebook
Sign up using Email and Password
Post as a guest
Required, but never shown
Required, but never shown
Required, but never shown
Required, but never shown
Required, but never shown
Required, but never shown
Required, but never shown
Required, but never shown
Required, but never shown